Đến nội dung

buitudong1998 nội dung

Có 841 mục bởi buitudong1998 (Tìm giới hạn từ 20-04-2020)



Sắp theo                Sắp xếp  

#585201 Tìm $k$ thỏa mãn $(2^a+3^a+4^a)(6^a+12^a+8^a)\le k24^...

Đã gửi bởi buitudong1998 on 26-08-2015 - 21:15 trong Bất đẳng thức và cực trị

Cho $a\in[1;2]$. Tìm $k$ tốt nhất để bất đẳng thức sau đúng $$(2^a+3^a+4^a)(6^a+12^a+8^a)\le k24^{a}.$$

-----

Bài toán trên xuất phát từ bài toán sau: Cho $a\in[1;2]$. CMR $(2^a+3^a+4^a)(6^a+12^a+8^a)< 24^{a+1}$.

Là một bài tập mình gặp trong quá trình tổng hợp tài liệu. Mình không thích các đánh giá trong bài tập này, nó không chặt nên mới đặt ra bài tập trên. Do BĐT mình kém nên chưa tìm ra được số $k$ tốt hơn và gửi lên đây các bạn đánh giá giúp.

Em nghĩ ta nên trung thành với đề cũ anh ạ, câu này trong đề thi thử của chuyên Hà Tĩnh và em cũng công nhận là nó quá lỏng

Thêm nữa, khi biến đổi: $BDT\Leftrightarrow (2^{a}+3^{a}+4^{a})(\frac{1}{2^{a}}+\frac{1}{3^{a}}+\frac{1}{4^{a}})\leqslant k\Leftrightarrow (\frac{2}{3})^{a}+(\frac{3}{2})^{a}+(\frac{3}{4})^{a}+(\frac{4}{3})^{a}+(\frac{2}{4})^{a}+(\frac{4}{2})^{2}\leqslant k-3$

Với $a\in \left [ 1;2 \right ]$ và dùng : $a^{x}\leqslant a$ khi  $a\leqslant 1; x\geqslant 1$

                                                    $a^{x}\geqslant a$ khi $a;x\geqslant 1$

Ta cũng tìm được $k\geqslant 12\tfrac{17}{18}$ rồi

Còn khi nhập biểu thức lên máy tính thì trên đoạn $1$ và $2$, nó đồng biến và em nghĩ $k\geqslant 12\tfrac{41}{144}$

Nhưng chứng minh điều này thực sự.......khó




#584785 Bất đẳng thức chuẩn bị cho kì thi THPTQG 2015-2016

Đã gửi bởi buitudong1998 on 24-08-2015 - 23:02 trong Bất đẳng thức và cực trị

Mở rộng của bài trên: Cho $x,y,z$ dương và $x+y+z=3$. Tìm giá trị nhỏ nhất của biểu thức:

$P=\frac{x}{2+\sqrt{2x^{2}+4x-5}}+\frac{y}{2+\sqrt{2y^{2}+4y-5}}+\frac{z}{2+\sqrt{2z^{2}+4z-5}}$




#584770 Bất đẳng thức chuẩn bị cho kì thi THPTQG 2015-2016

Đã gửi bởi buitudong1998 on 24-08-2015 - 22:35 trong Bất đẳng thức và cực trị

Bài ....: Chứng minh rằng: $x+\sqrt{2(x^2-x+1)} \geq 1+\sqrt{x}$   $\forall x \geq 0$

Xét trường hợp $x\geqslant 1+\sqrt{x}$ thì BĐT hiển nhiên đúng

Trường hợp còn lại: $x\leqslant 1+\sqrt{x}$, đặt $\sqrt{x}=t\geqslant 0$, chuyển vế bình phương và rút gọn ta được:

$BDT\Leftrightarrow t^{4}+2t^{3}-t^{2}-2t+1\geqslant 0\Leftrightarrow (t^{2}+t-1)^{2}\geqslant 0$ (Luôn đúng), vậy trong cả hai trường hợp ta đều có ĐPCM




#584161 $\frac{x+1}{x^{2}-x+2}+\frac...

Đã gửi bởi buitudong1998 on 22-08-2015 - 23:48 trong Phương trình - hệ phương trình - bất phương trình

Giải bất phương trình:

$\frac{x+1}{x^{2}-x+2}+\frac{x^{2}-x+1}{x+2}\geqslant 1+\frac{x\sqrt{x-1}}{4}$




#544501 Tìm $GTNN$ của $P=\frac{abc(a+b+c)}{ab+bc+...

Đã gửi bởi buitudong1998 on 16-02-2015 - 17:22 trong Bất đẳng thức và cực trị

Cho $a,b,c$ dương thỏa mãn: $\sum \frac{1}{a^{4}+1}=1$. Tìm $GTNN$ của $P=\frac{abc(a+b+c)}{ab+bc+ca}$




#544500 Tìm $GTNN$ của: $P=\sum \frac{a^{6}+b...

Đã gửi bởi buitudong1998 on 16-02-2015 - 17:19 trong Bất đẳng thức và cực trị

Cho $a,b,c$ dương thỏa mãn $abc=2\sqrt{2}$. Tìm $GTNN$ của: $P=\sum \frac{a^{6}+b^{6}}{a^{4}+b^{4}+a^{2}b^{2}}$




#542453 Tìm $m$ để phương trình có nghiệm thuộc đoạn $\left [ 0;...

Đã gửi bởi buitudong1998 on 31-01-2015 - 20:24 trong Phương trình - hệ phương trình - bất phương trình

Cho phương trình: $2cosx-2sinx+sin.cosx-m=0$. Tìm $m$ để phương trình có nghiệm thuộc đoạn $\left [ 0; \frac{\pi }{2} \right ]$ 




#542434 Tìm $GTNN$ của biểu thức: $P=\sum \frac{2}...

Đã gửi bởi buitudong1998 on 31-01-2015 - 17:12 trong Bất đẳng thức và cực trị

Cho $x,y,z>0$ thỏa mãn $x+y+z \leqslant 3$.Tìm $GTNN$ của biểu thức: $P=\sum \frac{2}{x^{3}}+\sum \frac{1}{x^{2}-xy+y^{2}}$




#533984 Tìm $GTNN$ của biểu thức: $P=\sqrt{5-4x}+2...

Đã gửi bởi buitudong1998 on 20-11-2014 - 21:13 trong Bất đẳng thức và cực trị

Cho các số thực $x,y,z$ thỏa mãn: $x^{2}+y^{2}+z^{2}=1$. Tìm $GTNN$ của biểu thức: $P=\sqrt{5-4x}+2\sqrt{4-2(x+y+z)}$




#522010 Tìm $GTNN$ và $GTLN$ của: $P=\frac{x^...

Đã gửi bởi buitudong1998 on 30-08-2014 - 21:36 trong Bất đẳng thức và cực trị

Cho $x,y$ thoả mãn: $(x^{2}+y^{2}+1)^{2}+3x^{2}y^{2}+1=4x^{2}+5y^{2}$. Tìm $GTNN$ và $GTLN$ của: 

$P=\frac{x^{2}+2y^{2}-3x^{2}y^{2}}{x^{2}+y^{2}+1}$




#522008 Tìm $GTNN$ của: $A=(xy+yz+2xz)^{2}-\frac{8...

Đã gửi bởi buitudong1998 on 30-08-2014 - 21:32 trong Bất đẳng thức và cực trị

Cho $x,y,z$ thoả mãn: $x^{2}+y^{2}+z^{2}=1$. Tìm $GTNN$ của:

$A=(xy+yz+2xz)^{2}-\frac{8}{3+xy+yz+2xz}$




#522007 Tìm $GTNN$ của: $P=\frac{16}{\sqrt...

Đã gửi bởi buitudong1998 on 30-08-2014 - 21:29 trong Bất đẳng thức và cực trị

Cho $x,y,z$ không âm thoả mãn: $x^{2}+y^{2}+z^{2}=3$. Tìm $GTNN$ của: 

$P=\frac{16}{\sqrt{\sum x^{2}y^{2}+1}}+\frac{xy+yz+zx+1}{x+y+z}$




#520218 $\left\{\begin{matrix} \sqrt{x...

Đã gửi bởi buitudong1998 on 18-08-2014 - 20:49 trong Phương trình - hệ phương trình - bất phương trình

Giải hệ phương trình: 

$\left\{\begin{matrix} \sqrt{x}+\sqrt[3]{y^{3}-1}=3 & & \\ x^{2}+y^{3}=82& & \end{matrix}\right.$




#520215 CMR: $x^{2}+4y^{2}< 1$

Đã gửi bởi buitudong1998 on 18-08-2014 - 20:46 trong Bất đẳng thức và cực trị

Cho các số thực dương $x,y$ thoả mãn: $x^{3}+y^{3}=x-y$. CMR: $x^{2}+4y^{2}< 1$ 




#520026 $$\sum \sqrt[3]{\frac{1}{a}...

Đã gửi bởi buitudong1998 on 17-08-2014 - 09:07 trong Bất đẳng thức - Cực trị

Cho $a,b,c >0$ và $ab+bc+ca=1$. Hãy chứng minh rằng:

$$\sqrt[3]{\frac{1}{a}+6b}+\sqrt[3]{\frac{1}{b}+6c}+\sqrt[3]{\frac{1}{c}+6a }\leq\frac{1}{abc}$$

Xem tại đây




#519927 $$\left\{\begin{matrix} z^3+z(x-y)^2...

Đã gửi bởi buitudong1998 on 16-08-2014 - 18:20 trong Phương trình, hệ phương trình và bất phương trình

lời giả đâu anh nhỉ

Nhấn vào số 1 ở đầu bài




#519916 $$\left\{\begin{matrix} z^3+z(x-y)^2...

Đã gửi bởi buitudong1998 on 16-08-2014 - 17:05 trong Phương trình, hệ phương trình và bất phương trình

Giải hệ phương trình:$$\left\{\begin{matrix} z^3+z(x-y)^2 = 16\\ y^3+y(z-x)^2 = 30\\ x^3+x(y-z)^2 = 2 \end{matrix}\right.$$

VMO 2004




#518788 CMR: $x+\sqrt{xy}+\sqrt[3]{xyz}\leqsl...

Đã gửi bởi buitudong1998 on 10-08-2014 - 13:15 trong Bất đẳng thức - Cực trị

Cho $x,y,z$ dương; $x+y+z=1$
CMR: $x+\sqrt{xy}+\sqrt[3]{xyz}\leqslant \frac{4}{3}$

$VT=x+\frac{1}{2}\sqrt{x.4y}+\frac{1}{4}\sqrt[3]{x.4y.16z}\leqslant \frac{4}{3}(x+y+z)=\frac{4}{3}$




#518694 $\sum \sqrt{a^{4}+a^{2}b^{2...

Đã gửi bởi buitudong1998 on 09-08-2014 - 21:34 trong Bất đẳng thức và cực trị

Cho $a,b,c$ dương Chứng minh các BDT:

a) $abc=8$. CMR: $\sum \frac{a^{2}}{\sqrt{(1+a^{3})(1+b^{3})}}\geqslant \frac{4}{3}$

b) $\sum \sqrt{a^{4}+a^{2}b^{2}+b^{4}}\geqslant \sum a\sqrt{2a^{2}+bc}$




#518692 CMR: $\frac{a}{a+bc}+\frac{b}...

Đã gửi bởi buitudong1998 on 09-08-2014 - 21:29 trong Bất đẳng thức và cực trị

Cho $a,b,c$ dương thoả mãn: $a+b+c=1$. CMR: $\frac{a}{a+bc}+\frac{b}{b+ca}+\frac{\sqrt{abc}}{c+ab}\geqslant 1+\frac{3\sqrt{3}}{4}$




#517226 $ln(x^{2}+x+1)+x+x^{4}=0$

Đã gửi bởi buitudong1998 on 02-08-2014 - 22:24 trong Phương trình - hệ phương trình - bất phương trình

Giải phương trình: $ln(x^{2}+x+1)+x+x^{4}=0$




#516977 $\left\{\begin{matrix} x^{3}(3y+...

Đã gửi bởi buitudong1998 on 01-08-2014 - 20:39 trong Phương trình - hệ phương trình - bất phương trình

Giải hệ phương trình:

$\left\{\begin{matrix} x^{3}(3y+8)=64 & & \\ x(y+1)(y^{2}+5y+7)=12+x& & \end{matrix}\right.$




#516975 $\left\{\begin{matrix} \sqrt{x+2...

Đã gửi bởi buitudong1998 on 01-08-2014 - 20:36 trong Phương trình - hệ phương trình - bất phương trình

Tìm $m$ để hệ phương trình sau có nghiệm:

$\left\{\begin{matrix} \sqrt{x+2}+\sqrt{y+2}=m+1 & & \\ \sqrt{x-2}+\sqrt{y-2}=1-2m& & \end{matrix}\right.$




#516766 $\left\{\begin{matrix} x^{2}+y^...

Đã gửi bởi buitudong1998 on 31-07-2014 - 21:29 trong Phương trình - hệ phương trình - bất phương trình

Giải hệ phương trình: 

$\left\{\begin{matrix} x^{2}+y^{2}+xy+1=4y & & \\ y(x+y)^{2}=2x^{2}+7y+2& & \end{matrix}\right.$




#516737 $\left\{\begin{matrix} xy+x+y=x^{2...

Đã gửi bởi buitudong1998 on 31-07-2014 - 20:18 trong Phương trình - hệ phương trình - bất phương trình

Giải hệ phương trình:

$\left\{\begin{matrix} xy+x+y=x^{2}-2y^{2} & & \\ x\sqrt{2y}-y\sqrt{x-1}=2x-2y& & \end{matrix}\right.$